Automobile Industry

This topic has expert replies
User avatar
Master | Next Rank: 500 Posts
Posts: 439
Joined: Sat Aug 09, 2008 8:32 am
Location: India
Thanked: 34 times
Followed by:28 members

Automobile Industry

by sivaelectric » Sun Jun 05, 2011 12:39 am
Recently the stock of a large automobile manufacturer rose when the company exceeded analyst' predictions of quarterly revenue by a significant margin. But exceeding predictions would not have caused the stock to rise had it not been for the introduction of a new line of popular vehicles. Therefore the manufacturer should continue to design popular new vehicles to increase its stock price.

Which of the following, if true, would most seriously weaken the conclusion about how to help the stock rise?
  • A. The manufacturer has plans to introduce one new line of vehicles every year.
    B. The new line of vehicles did not contribute to the quarter' total revenue figures.
    C. The value of the stock rose the entire year prior to exceeding analysts' predictions in the recent quarter.
    D. In past quarters where no new vehicle lines were introduced, exceeding analysts' revenue predictions frequently caused the stock to rise.
    E. When manufacturers introduce new vehicle lines, other factors in addition to exceeding revenue predictions sometimes cause their stock to rise.
Last edited by sivaelectric on Sun Jun 05, 2011 1:15 am, edited 1 time in total.
If I am wrong correct me :), If my post helped let me know by clicking the Thanks button ;).

Chitra Sivasankar Arunagiri

Junior | Next Rank: 30 Posts
Posts: 13
Joined: Tue May 17, 2011 9:27 am
GMAT Score:620

by chilledlife » Sun Jun 05, 2011 12:53 am
On exceeding analyst predictions the stock of the auto manufacture rose. However, introduction of the new line of popular vehicles was key.
Conclusions: Designing popular new vehicles will increase the company stock price.
We have to find a reason for increase in stock price other than the design of popular vehicles to weaken the conclusion.

A. The manufacturer has plans to introduce one new line of vehicles every year. Out of scope
B. The new line of vehicles did not contribute to the quarter' total revenue figures. Revenue increase + popular vehicles --> high stock price. No mention about the revenue of the new line of vehicles made. Out of Scope.
C. The value of the stock rose the entire year prior to exceeding analysts' predictions in the recent quarter. We do not know why the value of the stock rose. It could be due to the launch of the new line of popular cars or it could be any other reason. Out of Scope.
D. In past quarters where no new vehicle lines were introduced exceeding analysts' revenue predictions frequently caused the stock to rise. Out of Scope. This does not mean that the new line of cars will not help in the rise of stock price.
E. When manufacturers introduce new vehicle lines, other factors in addition to exceeding revenue predictions sometimes cause their stock to rise. Here new cars alone are not responsible for the rise in stock.

IMO E is the correct answer.

User avatar
Master | Next Rank: 500 Posts
Posts: 439
Joined: Sat Aug 09, 2008 8:32 am
Location: India
Thanked: 34 times
Followed by:28 members

by sivaelectric » Sun Jun 05, 2011 1:19 am
In addition to is the key word in that. Additional reasons are not asked for the stock rise. We should weaken the conclusion by saying that new vehicle release was not the reason for stocks raise. :)
If I am wrong correct me :), If my post helped let me know by clicking the Thanks button ;).

Chitra Sivasankar Arunagiri

User avatar
Legendary Member
Posts: 1309
Joined: Mon Apr 04, 2011 5:34 am
Location: India
Thanked: 310 times
Followed by:123 members
GMAT Score:750

by cans » Sun Jun 05, 2011 1:54 am
IMO E
If my post helped you- let me know by pushing the thanks button ;)

Contact me about long distance tutoring!
[email protected]

Cans!!

User avatar
Senior | Next Rank: 100 Posts
Posts: 37
Joined: Tue Jan 18, 2011 2:50 am
Thanked: 2 times

by abidshariff » Sun Jun 05, 2011 2:24 am
well..I guess it's D...the author is making a general statement "Therefore the manufacturer should continue to design popular new vehicles to increase its stock price"...what if in the previous quarters,the stock price did not rise, though the new vehicles were introduced.

E can't be the answer. watch out for "sometimes" in the sentence. So I will assume that most of the times, introduction of new vehicle lines had, indeed, led to the rise in the stocks, and so the statement, partly, supports the author's conclusion.

Moral: Watch out for such words as 'some', 'sometimes', they can be too dangerous in GMAT.

User avatar
Master | Next Rank: 500 Posts
Posts: 439
Joined: Sat Aug 09, 2008 8:32 am
Location: India
Thanked: 34 times
Followed by:28 members

by sivaelectric » Sun Jun 05, 2011 2:33 am
OA D
If I am wrong correct me :), If my post helped let me know by clicking the Thanks button ;).

Chitra Sivasankar Arunagiri

Master | Next Rank: 500 Posts
Posts: 102
Joined: Fri Nov 05, 2010 7:10 am
Thanked: 8 times
Followed by:2 members
GMAT Score:690

by gmatjeet » Sun Jun 05, 2011 2:50 am
IMHO, the answer should be D.

Question stem concludes that only new lines of cars will help the stock rise. To weaken we need an option which can help in stock rise.

User avatar
Legendary Member
Posts: 1101
Joined: Fri Jan 28, 2011 7:26 am
Thanked: 47 times
Followed by:13 members
GMAT Score:640

by HSPA » Sun Jun 05, 2011 3:27 am
This is a causal question and such questions are common in gmat.

conclusion: Rise in stock is due to new car model.
Cause: new car model
effect: stock rise

In order to weaken the conclusion we just need to say that rise in stock is due to some other reason.

D it is.
First take: 640 (50M, 27V) - RC needs 300% improvement
Second take: coming soon..
Regards,
HSPA.

User avatar
Master | Next Rank: 500 Posts
Posts: 407
Joined: Tue Jan 25, 2011 9:19 am
Thanked: 25 times
Followed by:7 members

by Ozlemg » Sun Jun 05, 2011 12:16 pm
my answer is D

totally agree with HSPA.
The more you suffer before the test, the less you will do so in the test! :)

Junior | Next Rank: 30 Posts
Posts: 21
Joined: Sun Jun 05, 2011 10:39 pm
Thanked: 2 times
Followed by:1 members

by worldpeace93 » Tue Jun 14, 2011 12:49 am
My answer is D

Junior | Next Rank: 30 Posts
Posts: 14
Joined: Sun Jun 06, 2010 8:21 am

by sainagalla » Tue Jun 14, 2011 3:35 am
showing that the effect exists in the absence of the cause will weaken the conclusion. this is best stated in option D.